Saltar al contenido principal
LibreTexts Español

1.11: Números primos

  • Page ID
    117381
  • \( \newcommand{\vecs}[1]{\overset { \scriptstyle \rightharpoonup} {\mathbf{#1}} } \) \( \newcommand{\vecd}[1]{\overset{-\!-\!\rightharpoonup}{\vphantom{a}\smash {#1}}} \)\(\newcommand{\id}{\mathrm{id}}\) \( \newcommand{\Span}{\mathrm{span}}\) \( \newcommand{\kernel}{\mathrm{null}\,}\) \( \newcommand{\range}{\mathrm{range}\,}\) \( \newcommand{\RealPart}{\mathrm{Re}}\) \( \newcommand{\ImaginaryPart}{\mathrm{Im}}\) \( \newcommand{\Argument}{\mathrm{Arg}}\) \( \newcommand{\norm}[1]{\| #1 \|}\) \( \newcommand{\inner}[2]{\langle #1, #2 \rangle}\) \( \newcommand{\Span}{\mathrm{span}}\) \(\newcommand{\id}{\mathrm{id}}\) \( \newcommand{\Span}{\mathrm{span}}\) \( \newcommand{\kernel}{\mathrm{null}\,}\) \( \newcommand{\range}{\mathrm{range}\,}\) \( \newcommand{\RealPart}{\mathrm{Re}}\) \( \newcommand{\ImaginaryPart}{\mathrm{Im}}\) \( \newcommand{\Argument}{\mathrm{Arg}}\) \( \newcommand{\norm}[1]{\| #1 \|}\) \( \newcommand{\inner}[2]{\langle #1, #2 \rangle}\) \( \newcommand{\Span}{\mathrm{span}}\)\(\newcommand{\AA}{\unicode[.8,0]{x212B}}\)

    Definición\(\PageIndex{1}\)

    Un entero\(p\) es primo si\(p\ge 2\) y los únicos divisores positivos de\(p\) son\(1\) y\(p\). Un entero\(n\) es compuesto si\(n\ge 2\) y no\(n\) es primo.

    Observación\(\PageIndex{1}\)

    El número no\(1\) es primo ni compuesto.

    Lema\(\PageIndex{1}\)

    Un entero\(n\ge 2\) es compuesto si y sólo si hay enteros\(a\) y\(b\) tal que\(n=ab\),\(1<a<n\), y\(1<b<n\).

    Prueba

    Vamos\(n\ge 2\). Si\(n\) es compuesto hay un entero positivo\(a\) tal que\(a\ne 1\),\(a\ne n\) y\(a\mid n\). Esto significa que\(n=ab\) para algunos\(b\). Desde\(n\) y\(a\) son positivos así es\(b\). De ahí\(1\le a\) y\(1\le b\). Por Teorema 1.4.1 (10)\(a\le n\) y\(b\le n\). Desde\(a\ne 1\) y\(a\ne n\) tenemos\(1<a<n\). Si\(b=1\) entonces\(a=n\), lo cual no es posible, entonces\(b\ne 1\). Si\(b=n\) entonces\(a=1\), lo cual tampoco es posible. Entonces\(1<b<n\). Lo contrario es cierto por la definición de números compuestos.

    Lema\(\PageIndex{2}\)

    Si\(n>1\), hay un prime\(p\) tal que\(p\mid n\).

    Prueba

    Supongamos que hay algún número entero\(n>1\) que no tiene divisor primo. Dejar\(S\) denotar el conjunto de todos esos enteros. Por el Principio de Ordenamiento del Bien existe un número entero más pequeño, llámalo\(m\). Ahora\(m>1\) y no tiene divisor primo. Entonces\(m\) no puede ser prime. De ahí\(m\) que sea compuesto. Por lo tanto por Lemma\(\PageIndex{1}\)\[m=ab,\quad 1<a<m,\quad 1<b<m.\nonumber \] Desde\(1<a<m\) entonces no\(a\) está en el set\(S\). Entonces\(a\) debe tener un divisor primo, llámalo\(p\). Entonces\(p\mid a\) y\(a\mid m\) así por Teorema 1.4.1,\(p\mid m\). Esto contradice el hecho de que no\(m\) tenga divisor primo. Entonces el conjunto\(S\) debe estar vacío y esto prueba el lema.

    Teorema \(\PageIndex{1}\): Euclid's Theorem\(^{1}\)

    Hay infinitamente muchos números primos.

    Prueba

    Supongamos, a modo de contradicción, que sólo hay un número finito de números primos, digamos:\[p_1,p_2,\dotsc,p_n.\nonumber \] Definir\[N=p_1p_2\dotsm p_n+1.\nonumber \] Desde\(p_1\ge 2\), claramente\(N\ge 3\). Entonces por Lemma 10.2\(N\) tiene un divisor primo\(p\). Por suposición\(p=p_i\) para algunos\(i=1,\dotsc,n\). Vamos\(a=p_1\dotsm p_n\). Tenga en cuenta que\[a=p_i\left(p_1p_2\dotsm p_{i-1}p_{i+1}\dotsm p_n\right),\nonumber \] así\(p_i\mid a\). Ahora\(N=a+1\) y por suposición\(p_i\mid (a+1)\). Entonces por el Ejercicio 1.4.3\(p_i\mid ((a+1)-a)\), es decir\(p_i\mid 1\). Por Axioma Básico 3 en la Sección 1.2 esto implica que\(p_i=1\). Esto contradice el hecho de que los primos son\(>1\). De ello se deduce que la suposición de que sólo hay finitamente muchos primos no es cierta.

    Teorema \(\PageIndex{2}\)

    Si\(n>1\) es compuesto entonces\(n\) tiene un divisor primo\(p\le\sqrt{n}\).

    Prueba

    Dejar\(n>1\) ser compuesto. Entonces\(n=ab\) donde\(1<a<n\) y\(1<b<n\). Afirmamos que uno de\(a\) o\(b\) es\(\le\sqrt{n}\). Si no, entonces\(a>\sqrt{n}\) y\(b>\sqrt{n}\). De ahí\(n=ab>\sqrt{n}\,\sqrt{n}=n\). Esto implica eso\(n>n\), una contradicción. Entonces\(a\le\sqrt{n}\) o\(b\le\sqrt{n}\). Supongamos\(a\le\sqrt{n}\). Ya que\(1<a\), por Lemma\(\PageIndex{2}\) hay un prime\(p\) tal que\(p\mid a\). De ahí, por Teorema 1.4.1 ya que\(a\mid n\) tenemos\(p\mid n\). También por Teorema 1.4.1 ya\(p\mid a\) que tenemos\(p\le a\le\sqrt{n}\).

    La prueba del teorema de Euclides insinúa una forma de generar un nuevo número primo, dada una lista de números primos ya conocidos. \(^{2}\)Sin embargo, este método no produce todos los primos en el orden en que ocurren entre los enteros. Una forma más sistemática de producir primes, conocida como el tamiz de Eratóstenes. \(^{3}\)El tamiz de Eratóstenes funciona de esta manera:

    Para generar todos los números primos menores o iguales a\(n\), comenzamos por escribir todos los enteros positivos desde\(2\) hasta\(n\). Por ejemplo, si deseamos encontrar todos los números primos que son como máximo 100, comenzamos por escribir

      2 3 4 5 6 7 8 9 10
    11 12 13 14 15 16 17 18 19 20
    21 22 23 24 25 26 27 28 29 30
    31 32 33 34 35 36 37 38 39 40
    41 42 43 44 45 46 47 48 49 50
    51 52 53 54 55 56 57 58 59 60
    61 62 63 64 65 66 67 68 69 70
    71 72 73 74 75 76 77 78 79 80
    81 82 83 84 85 86 87 88 89 90
    91 92 93 94 95 96 97 98 99 100

    (aquí la disposición en forma de rejilla que hemos utilizado es conveniente pero no necesaria para que funcione el tamiz).

    A continuación aplicamos repetidamente el siguiente paso:

    Encierra en círculo el primer número no tachado; luego tachar cualquier otro múltiplo de este número que aparezca en la lista que aún no haya sido tachado.

    Repetimos este proceso hasta que no podamos, es decir, hasta que no haya más números no tachados para rodear. Los números primos son entonces los números que han sido enmarcados en un círculo.

    Por ejemplo, en nuestro ejemplo anterior comenzaríamos dando vueltas al 2, el primer número que no ha sido dado en un círculo ni tachado. Entonces tacharíamos todos los múltiplos de 2 en la lista que no fueran 2. Esto produce la siguiente figura:

      \(\fbox{2}\) 3 \(\xcancel{4}\) 5 \(\xcancel{6}\) 7 \(\xcancel{8}\) 9 \(\xcancel{10}\)
    11 \(\xcancel{12}\) 13 \(\xcancel{14}\) 15 \(\xcancel{16}\) 17 \(\xcancel{18}\) 19 \(\xcancel{20}\)
    21 \(\xcancel{22}\) 23 \(\xcancel{24}\) 25 \(\xcancel{26}\) 27 \(\xcancel{28}\) 29 \(\xcancel{30}\)
    31 \(\xcancel{32}\) 33 \(\xcancel{34}\) 35 \(\xcancel{36}\) 37 \(\xcancel{38}\) 39 \(\xcancel{40}\)
    41 \(\xcancel{42}\) 43 \(\xcancel{44}\) 45 \(\xcancel{46}\) 47 \(\xcancel{48}\) 49 \(\xcancel{50}\)
    51 \(\xcancel{52}\) 53 \(\xcancel{54}\) 55 \(\xcancel{56}\) 57 \(\xcancel{58}\) 59 \(\xcancel{60}\)
    61 \(\xcancel{62}\) 63 \(\xcancel{64}\) 65 \(\xcancel{66}\) 67 \(\xcancel{68}\) 69 \(\xcancel{70}\)
    71 \(\xcancel{72}\) 73 \(\xcancel{74}\) 75 \(\xcancel{76}\) 77 \(\xcancel{78}\) 79 \(\xcancel{80}\)
    81 \(\xcancel{82}\) 83 \(\xcancel{84}\) 85 \(\xcancel{86}\) 87 \(\xcancel{88}\) 89 \(\xcancel{90}\)
    91 \(\xcancel{92}\) 93 \(\xcancel{94}\) 95 \(\xcancel{96}\) 97 \(\xcancel{98}\) 99 \(\xcancel{100}\)

    El siguiente paso es hacer un círculo 3 (el primer número que no esté ya en círculo o tachado) y tachar cualquier múltiplo de 3 (como el 9, 15, etc.) que no esté ya tachado. Cuando este paso está terminado, nuestra lista se ve así:

      \(\fbox{2}\) \(\fbox{3}\) \(\xcancel{4}\) 5 \(\xcancel{6}\) 7 \(\xcancel{8}\) \(\xcancel{9}\) \(\xcancel{10}\)
    11 \(\xcancel{12}\) 13 \(\xcancel{14}\) \(\xcancel{15}\) \(\xcancel{16}\) 17 \(\xcancel{18}\) 19 \(\xcancel{20}\)
    \(\xcancel{21}\) \(\xcancel{22}\) 23 \(\xcancel{24}\) 25 \(\xcancel{26}\) \(\xcancel{27}\) \(\xcancel{28}\) 29 \(\xcancel{30}\)
    31 \(\xcancel{32}\) \(\xcancel{33}\) \(\xcancel{34}\) 35 \(\xcancel{36}\) 37 \(\xcancel{38}\) \(\xcancel{39}\) \(\xcancel{40}\)
    41 \(\xcancel{42}\) 43 \(\xcancel{44}\) \(\xcancel{45}\) \(\xcancel{46}\) 47 \(\xcancel{48}\) 49 \(\xcancel{50}\)
    \(\xcancel{51}\) \(\xcancel{52}\) 53 \(\xcancel{54}\) 55 \(\xcancel{56}\) \(\xcancel{57}\) \(\xcancel{58}\) 59 \(\xcancel{60}\)
    61 \(\xcancel{62}\) \(\xcancel{63}\) \(\xcancel{64}\) 65 \(\xcancel{66}\) 67 \(\xcancel{68}\) \(\xcancel{69}\) \(\xcancel{70}\)
    71 \(\xcancel{72}\) 73 \(\xcancel{74}\) \(\xcancel{75}\) \(\xcancel{76}\) 77 \(\xcancel{78}\) 79 \(\xcancel{80}\)
    \(\xcancel{81}\) \(\xcancel{82}\) 83 \(\xcancel{84}\) 85 \(\xcancel{86}\) \(\xcancel{87}\) \(\xcancel{88}\) 89 \(\xcancel{90}\)
    91 \(\xcancel{92}\) \(\xcancel{93}\) \(\xcancel{94}\) 95 \(\xcancel{96}\) 97 \(\xcancel{98}\) \(\xcancel{99}\) \(\xcancel{100}\)

    Si seguimos hasta que todos los números hayan sido rodeados o tachados, nuestra lista se verá así:

      \(\fbox{2}\) \(\fbox{3}\) \(\xcancel{4}\) \(\fbox{5}\) \(\xcancel{6}\) \(\fbox{7}\) \(\xcancel{8}\) \(\xcancel{9}\) \(\xcancel{10}\)
    \(\fbox{11}\) \(\xcancel{12}\) \(\fbox{13}\) \(\xcancel{14}\) \(\xcancel{15}\) \(\xcancel{16}\) \(\fbox{17}\) \(\xcancel{18}\) \(\fbox{19}\) \(\xcancel{20}\)
    \(\xcancel{21}\) \(\xcancel{22}\) \(\fbox{23}\) \(\xcancel{24}\) \(\xcancel{25}\) \(\xcancel{26}\) \(\xcancel{27}\) \(\xcancel{28}\) \(\fbox{29}\) \(\xcancel{30}\)
    \(\fbox{31}\) \(\xcancel{32}\) \(\xcancel{33}\) \(\xcancel{34}\) \(\xcancel{35}\) \(\xcancel{36}\) \(\fbox{37}\) \(\xcancel{38}\) \(\xcancel{39}\) \(\xcancel{40}\)
    \(\fbox{41}\) \(\xcancel{42}\) \(\fbox{43}\) \(\xcancel{44}\) \(\xcancel{45}\) \(\xcancel{46}\) \(\fbox{47}\) \(\xcancel{48}\) \(\xcancel{49}\) \(\xcancel{50}\)
    \(\xcancel{51}\) \(\xcancel{52}\) \(\fbox{53}\) \(\xcancel{54}\) \(\xcancel{55}\) \(\xcancel{56}\) \(\xcancel{57}\) \(\xcancel{58}\) \(\fbox{59}\) \(\xcancel{60}\)
    \(\fbox{61}\) \(\xcancel{62}\) \(\xcancel{63}\) \(\xcancel{64}\) \(\xcancel{65}\) \(\xcancel{66}\) \(\fbox{67}\) \(\xcancel{68}\) \(\xcancel{69}\) \(\xcancel{70}\)
    \(\fbox{71}\) \(\xcancel{72}\) \(\fbox{73}\) \(\xcancel{74}\) \(\xcancel{75}\) \(\xcancel{76}\) \(\xcancel{77}\) \(\xcancel{78}\) \(\fbox{79}\) \(\xcancel{80}\)
    \(\xcancel{81}\) \(\xcancel{82}\) \(\fbox{83}\) \(\xcancel{84}\) \(\xcancel{85}\) \(\xcancel{86}\) \(\xcancel{87}\) \(\xcancel{88}\) \(\fbox{89}\) \(\xcancel{90}\)
    \(\xcancel{91}\) \(\xcancel{92}\) \(\xcancel{93}\) \(\xcancel{94}\) \(\xcancel{95}\) \(\xcancel{96}\) \(\fbox{97}\) \(\xcancel{98}\) \(\xcancel{99}\) \(\xcancel{100}\)

    Observe cómo el nombre de esta técnica es descriptivo, al tachar todos los múltiplos de los números en un círculo, dejamos que estos números se nos escapen como si fueran a través de un tamiz, mientras podemos imaginar que los primos en círculo quedan atrapados en el tamiz. En nuestro ejemplo, los números conservados por el tamiz de Eratóstenes, es decir, los números primos menores que\(100\), son los números\[2,3,5,7,11,13,17,19,23,29,31,37,41,43,47,53,59,61,67,71,73,79,83,89,97.\nonumber \]

    Definición\(\PageIndex{2}\)

    La primalidad de un entero\(n \geq 2\) se refiere a su pertenencia (o no) en el conjunto de números primos. Verificar la primalidad de los\(n\) medios para determinar si\(n\) es primo.

    Observación\(\PageIndex{2}\)

    Podemos usar Teorema\(\PageIndex{2}\) para ayudar a verificar la primalidad de un entero: Para comprobar si\(n>1\) es primo o no solo necesitamos tratar de dividirlo por todos los primos\(p\le\sqrt{n}\). Si ninguno de estos primos se divide\(n\) entonces\(n\) debe ser primo.

    Ejemplo \(\PageIndex{1}\)

    Considera el número\(97\). Tenga en cuenta que\(\sqrt{97}<\sqrt{100}=10\). Los primos\(\le 10\) son\(2\),\(3\),\(5\), y\(7\). Uno comprueba fácilmente que\(97\bmod 2=1\),\(97\bmod 3=1\),\(97\bmod 5=2\),\(97\bmod 7=6\). Entonces ninguno de los primos\(2\),,\(3\)\(5\),\(7\) divide\(97\) y\(97\) es primo por teorema\(\PageIndex{2}\).

    Esto nos da un ligero atajo para encontrar primos con el Tamiz de Eratóstenes: en nuestro ejemplo anterior, una vez que hemos dado un círculo 7 y tachado sus múltiplos en el ejemplo anterior, cada otro número actualmente en la lista que aún no haya sido marcado o tachado está garantizado para ser primo y puede inmediatamente ser encerrado en un círculo, ya que 7 es el número primo más grande que es menor o igual a la raíz cuadrada de 100.

    Definición\(\PageIndex{3}\)

    Vamos\(x\in\mathbb{R}\),\(x>0\). La función de recuento de primos\(\pi(x)\) se define como el número de primos\(p\) tal que\(p\le x\).

    Por ejemplo, ya que los únicos primos\(p\le 10\) son 2, 3, 5 y 7 tenemos\(\pi(10) = 4\).

    Aquí hay una tabla de valores de\(\pi(10^i)\) for\(i=2, \dots, 10\). También incluimos aproximaciones conocidas a\(\pi(x)\). Tenga en cuenta que las fórmulas para las aproximaciones no dan valores enteros, sino que para la tabla hemos redondeado cada una al entero más cercano. Los valores de la tabla se calcularon utilizando el sistema de álgebra computacional Maple. \[\left | \begin {array}{rrrr} x & \pi(x) & \frac x{\ln(x)} & \int_2^x\frac{1}{\ln(t)}dt \\ & & & \\ 10^2 & 25 & 22 & 29\\ 10^3 & 168& 145& 177\\ 10^4 & 1229 & 1086 & 1245\\ 10^5 & 9592 & 8686 & 9629\\ 10^6 & 78498 & 72382 & 78627\\ 10^7 & 664579 & 620421 & 664917\\ 10^8 & 5761455 & 5428681 & 5762208\\ 10^9 & 50847534 & 48254942 & 50849234\\ 10^{10} & 455052511 & 434294482 & 455055614 \end {array} \right|\nonumber \]Puedes juzgar por ti mismo qué aproximaciones parecen ser las mejores. Esta mesa se ha continuado \(^{4}\)hasta\(10^{28}\), pero sin duda la gente sigue trabajando para encontrar el valor de\(\pi(10^{n})\) para mayores\(n\). Por supuesto, las aproximaciones son fáciles de calcular con Maple o una herramienta computacional similar, pero el valor exacto de\(\pi(10^{28})\) es difícil de encontrar.

    ¿Por qué las funciones definidas por\(\frac{x}{\ln x}\) y\(\int_2^x \frac{dt}{\ln t}\) hacen tan bien como lo hacen en la aproximación\(\pi(x)\)? Esta no es una pregunta fácil de responder. La relación entre\(\pi(x)\) y\(\int_2^{x}\frac{dt}{\ln t}\) fue conjeturada por Gauss en\(1793\). Más de 100 años después, Hadamard y de la Vallée Poussin, independientemente entre sí, completaron con éxito pruebas de este resultado. Aquí hay una declaración equivalente.

    Teorema\(\PageIndex{3}\): The Prime Number Theorem

    \[\pi(x)\sim\frac x{\ln(x)} \quad \mbox{ for $x>0$}.\nonumber \]

    Observación\(\PageIndex{3}\)

    El\(\sim\) en el teorema del número primo significa que\[\lim_{x\to\infty}\frac{\pi(x)}{\frac x{\ln(x)}}=1.\nonumber \] Para una conexión entre\(\frac{x}{\ln x}\) y\(\int_2^x \frac{dt}{\ln t}\), ver Ejercicio\(\PageIndex{11}\).

    La distribución de los números primos entre los enteros es en algunos aspectos todavía misteriosa. Aunque hay infinitamente muchos primos, hay largos tramos de enteros consecutivos que no contienen primos.

    Teorema\(\PageIndex{4}\)

    Para cualquier entero positivo\(n\) hay un entero\(a\) tal que los enteros\(n\) consecutivos\[a, \; a+1, \; a+2, \; \dotsc, \; a+(n-1)\nonumber \] son todos compuestos.

    Prueba

    Dado\(n\ge 1\) let\(a=(n+1)!+2\). (Las factoriales se definieron en el Ejercicio 1.3.5.) Afirmamos que todos los números\[a, \; a+1, \; a+2, \; \dotsc, \; a+(n-1)\nonumber \] son compuestos. Ya que\((n+1)\ge 2\) claramente\(2\mid (n+1)!\) y\(2\mid 2\). De ahí\(2\mid ((n+1)!+2)\). Ya que\((n+1)!+2>2\),\((n+1)!+2\) es compuesto. Considera\[a+i=(n+1)!+i+2\nonumber \] dónde\(0\le i\le n-1\) es así\(2\le i+2\le n+1\). Así\(i+2\mid (n+1)!\) y\(i+2\mid (i+2).\) Por lo tanto\(i+2\mid (a+i)\). Ahora\(a+i>i+2>1\), así\(a+i\) es compuesto.

    Por otro lado, declaraciones como la Conjetura de Twin Prime describen cuán corta podría garantizarse que sería una lista de números compuestos consecutivos. Los primos gemelos son números primos que difieren exactamente en 2. Ejemplos de “primos gemelos” incluyen 5 y 7, 29 y 31, y 1427 y 1429. Entre primos gemelos, solo hay un número compuesto, pero ¿los primos gemelos eventualmente dejan de suceder?

    La conjetura de Twin Prime

    Hay infinitamente muchos pares de primos gemelos.

    Aunque esta conjetura no se ha probado, en abril de 2013 el matemático Yitang Zhang anunció una prueba de que hay infinitamente muchos pares de primos que difieren en 70 millones o menos. \(^{5}\)Al dar seguimiento a estas ideas, un año después The Polymath Project \(^{6}\)mejoró el resultado para mostrar que hay infinitamente muchos pares de primos que difieren en no más de 246.

    Otra conjetura que tiene vínculos con las ubicaciones de los números primos es la Conjetura de Goldbach.

    Conjetura de Goldbach

    Cada entero par\(n>2\) es la suma de dos primos.

    Esta conjetura surgió en correspondencia de 1742 entre Christian Goldbach y Leonhard Euler. Ha sido verificado por T. Oliveira e Silva, S. Herzog y S. Pardi\(^{7}\) para todos los enteros pares no mayores que\(4 \cdot 10^{18}\) pero permanece sin probar.

    Incluso existe una conexión entre la distribución de los números primos (en particular, qué tan aproximada de aproximación son para las funciones en el Teorema de los números primos\(\pi(x)\)) y la pregunta que quizás es más famosa entre todos los problemas actualmente sin resolver de las matemáticas: es el Riemann Hipótesis ¿verdad? No discutiremos la Hipótesis de Riemann aquí, pero se puede leer sobre ella (y su conexión con aproximaciones de\(\pi(x)\)) con un poco de investigación. \(^{8}\)

    Hemos cubierto mucho terreno en este capítulo. Verdaderamente, incluso miles de años después de que Euclides y Eratóstenes los estudiaran, los números primos siguen siendo una rica fuente de misterios comprensibles pero aún sin resolver.

    Ejercicios

    Ejercicio\(\PageIndex{1}\)

    Demostrar que\(2\) es el único número primo par. (Broma: De ahí que se diga que 2 es el prime “más extraño”.)

    Ejercicio \(\PageIndex{2}\)

    Mostrar que cada número primo que no sea 2 o 3 es uno menor que, o uno más de, un múltiplo de 6. (El Apéndice A organiza los primos menos de 200 en casos basados en este hecho.)

    Ejercicio \(\PageIndex{3}\)

    Demostrar que si\(p\) y\(q\) son primos y\(p\mid q\), entonces\(p=q\).

    Ejercicio\(\PageIndex{4}\)

    Usa la idea de la prueba del Teorema (Teorema\(\PageIndex{1}\)) de Euclides para demostrar que si\(q_1,q_2,\dotsc,q_n\) hay primos (no necesariamente los primos más pequeños, o consecutivos), entonces hay un primo\(q\notin\left\{q_1,\dotsc,q_n\right\}\).

    (Pista: Tomar\(N=q_1\dotsm q_n+1\); por Lemma\(\PageIndex{2}\) hay un prime\(q\) tal que\(q\mid N\). \(q\notin\left\{q_1,\dotsc,q_n\right\}\)Demuéstralo.)

    Ejercicio\(\PageIndex{5}\)

    Dejar\(p_1=2,p_2=3,p_3=5,\dotsc\) y, en general,\(p_i=\text{the }i\) -th prime. Demostrar o desacreditar que\[p_1p_2\dotsm p_n+1\nonumber \] es primordial para todos\(n\ge 1\).

    (Pista: Si\(n=1\) tenemos\(2+1=3\) es primo. Si\(n=2\) tenemos\(2\cdot 3+1=7\) es prime. Si\(n=3\) tenemos\(2\cdot 3\cdot 5+1=31\) es prime. Pruebe los siguientes valores de\(n\). Es posible que desee utilizar el Teorema\(\PageIndex{2}\) para verificar la primalidad.)

    Ejercicio\(\PageIndex{6}\)

    Al usar Teorema\(\PageIndex{2}\) como en Ejemplo\(\PageIndex{1}\), determinar la primalidad de los siguientes enteros:\[143,\quad 221,\quad 199,\quad 223,\quad 3521.\nonumber \]

    Ejercicio\(\PageIndex{7}\)
    1. Imitando la prueba del Teorema\(\PageIndex{2}\), muestran que si\(n=abc\) donde\(a>1\)\(b>1\),\(c>1\), y, entonces uno de\(a,b,c\) es menor o igual a\(\sqrt[3]{n}\).
    2. Comparando Teorema\(\PageIndex{2}\) y parte (a), llegar a una generalización para cuando\(n\) es el producto de\(k\) enteros, cada uno mayor que 1.
    Ejercicio\(\PageIndex{8}\)

    Usa el tamiz de Eratóstenes para encontrar todos los números primos menores a 300, mostrando tu trabajo.

    Ejercicio\(\PageIndex{9}\)

    Demostrar que mientras el tamiz de Eratóstenes se realiza en una lista\(2,\dots,n\) de enteros, cuando\(k\) se da un círculo a un número, el primer múltiplo de\(k\) ese entonces necesita ser tachado (porque no se ha tachado ya) es\(k^2\).

    Ejercicio\(\PageIndex{1}\)

    Utilice el teorema de números primos y una calculadora para aproximar el número de primos\(\le 10^8\). Nota\(\ln (10^8 )=8 \ln (10)\).

    Ejercicio \(\PageIndex{11}\)

    Usa la regla de L'hospital y el Teorema Fundamental del Cálculo (con integración por partes) para mostrar eso\[\lim_{x \to \infty} \frac{\int_2^{x}\frac{dt}{\ln t}}{\frac{x}{\ln x}} = 1,\nonumber \] y usar esto y Teorema\(\PageIndex{3}\) para demostrar que\[\pi(x) \sim \int_2^{x}\frac{dt}{\ln t}.\nonumber \]

    Ejercicio\(\PageIndex{12}\)

    Encuentra números compuestos\(10\) consecutivos.

    Ejercicio\(\PageIndex{13}\)
    1. \(2^n-1\)Siempre es primo si\(n\ge 2\)? Explique.
    2. ¿\(2^n-1\)Siempre es prime si\(n\) es prime? Explique.

    Notas al pie

    [1] Este teorema aparece como Proposición 20 en el Libro IX de los Elementos de Euclides, por lo que una prueba de este hecho se ha transmitido desde hace más de 2000 años.

    [2] Si aplicamos repetidamente la prueba del Teorema de Euclides, comenzando con\(p_1 = 2\) y dejando cada vez que\(p_{n+1}\) sea el divisor primo más pequeño cuando\(N\) se crea a partir de\(p_1,\ldots ,p_n\), entonces el orden en el que se generan los números primos\(2, 3, 7, 43\), comenzando con, etc., no está bien entendido. Para algunas notas sobre este orden de primos, visite la entrada en él en la Enciclopedia en línea de secuencias enteras: ver http://oeis.org/A000945.

    [3] Este método fue atribuido por un escritor antiguo al matemático griego Eratóstenes, quien vivió en el siglo III a.C. (también en Alejandría, aunque quizás un poco después de Euclides).

    [4] El valor\(\pi (10^{28})\) fue anunciado para ser\(157,589,269,275,973,410,412,739,598\) por David Baugh y Kim Walisch en agosto de 2020 (ver https://mersenneforum.org/showthread.php?p=555442).

    [5] Para un video atractivo sobre el avance, vea “Brechas entre Primes” de Numberphile en https://www.youtube.com/watch?v=vkMXdShDdtY.

    [6] https://polymathprojects.org/

    [7] “Verificación empírica de la conjetura de Goldbach par y cómputo de brechas primarias hasta”\(4\cdot 10^{18}\), Matemáticas de la Computación, Vol. 83, núm. 288 (julio de 2014), pp. 2033-2060.

    [8] Una fuente para consultar está en https://www.simonsfoundation.org/202...nn-hypothesis/.


    This page titled 1.11: Números primos is shared under a CC BY-NC-SA 4.0 license and was authored, remixed, and/or curated by Mike Barrus & W. Edwin Clark.